Diễn Đàn MathScopeDiễn Đàn MathScope
  Diễn Đàn MathScope
Ghi Danh Hỏi/Ðáp Thành Viên Social Groups Lịch Ðánh Dấu Ðã Ðọc

Go Back   Diễn Đàn MathScope > Sơ Cấp > Việt Nam và IMO > 2011

News & Announcements

Ngoài một số quy định đã được nêu trong phần Quy định của Ghi Danh , mọi người tranh thủ bỏ ra 5 phút để đọc thêm một số Quy định sau để khỏi bị treo nick ở MathScope nhé !

* Nội quy MathScope.Org

* Một số quy định chung !

* Quy định về việc viết bài trong diễn đàn MathScope

* Nếu bạn muốn gia nhập đội ngũ BQT thì vui lòng tham gia tại đây

* Những câu hỏi thường gặp

* Về việc viết bài trong Box Đại học và Sau đại học


Trả lời Gởi Ðề Tài Mới
 
Ðiều Chỉnh Xếp Bài
Old 10-01-2011, 05:22 PM   #1
n.v.thanh
Moderator
 
n.v.thanh's Avatar
 
Tham gia ngày: Nov 2009
Bài gởi: 2,849
Thanks: 2,980
Thanked 2,537 Times in 1,008 Posts
Kỳ Thi Chọn HSGQG Năm 2011 - Môn Toán Học - Đề Thi, Đáp Án và Kết Quả


Như các bạn đã biết, ngày 11 và 12 tháng 1 năm 2011, Bộ giáo dục tổ chức kì thi chọn học sinh giỏi quốc gia năm 2011. Để tránh tình trạng nhiều topic,loãng diễn đàn không cần thiết và tiện cho việc thảo luận giữa các thành viên, thay mặt Ban Quản Trị, nvthanh1994 lập ra topic này. Đây sẽ là nơi thảo luận trong suốt kì thi VMO 2011. Ngày mai và ngày kia anh Novae (Tức anh Minh) đưa đề thi VMO 2011 tức đề quốc gia môn Toán lên theo hình thức từ bài 1 tới bài 7 thay vì 2 topic cho ngày 1 ngày 2 hoặc 1 topic mà có 2 bài 1. Mathscope có những yêu cầu sau:

Trích:
  • Mọi người tuyệt đối không lập bất cứ một topic nào khác để bàn luận cùng chủ đề với topic này,nếu có thì sẽ ngay lập tức bị đóng lại .
  • Biết rằng thi xong ai cũng muốn bày tỏ cảm xúc của mình nhưng mong mọi người hãy bình tĩnh một lát trong khi anh Novae đang cấp tốc soạn đề, sau khi có đề thì mọi người thì thoải mái thảo luận, tất nhiên phải có văn hóa, hợp với quy định của Mathscope và không lạc đề rồi.
Sau khi có kết quả cũng như đáp án, ban quản trị sẽ ngay lập tức cung cấp thông tin vào topic này cho tiện việc tra cứu của thành viên. Sau khi kì thi kết thúc sẽ có một topic mới gồm có:

Trích:
VMO 2011:
  • Preparation(mấy cái đề Prepare của thầy Dũng,đề thi chon các trường các tỉnh
  • Danh sách học sinh tham dự.
  • Đề thi,lời giải,đáp án(bộ năm nay có công bố đáp án)
  • Kết quả.


làm kỉ niệm và tiếp tục cho kì thi TST không lâu sau đó.



Mong rằng VMO năm nay sẽ diễn ra tốt

đẹp cho toàn thể học sinh tham dự kì thi và cả Mathscope!
Thân.
T

[RIGHT][I][B]Nguồn: MathScope.ORG[/B][/I][/RIGHT]
 

thay đổi nội dung bởi: novae, 10-01-2011 lúc 09:08 PM
n.v.thanh is offline   Trả Lời Với Trích Dẫn
The Following 21 Users Say Thank You to n.v.thanh For This Useful Post:
binladen93 (11-01-2011), cattuong (11-01-2011), cuong_kimvan (11-01-2011), Curi_Gem (11-01-2011), death_vs_love (06-03-2011), ducadongnoi (05-03-2011), haimap27 (14-01-2011), huynhcongbang (12-01-2011), Ino_chan (06-03-2011), je.triste (25-02-2011), lady_kom4 (10-01-2011), linh_kc (20-02-2011), lk_95 (11-01-2011), long_chau2010 (11-01-2011), luatdhv (10-01-2011), lucatony1234 (13-01-2011), Member_Of_AMC (12-01-2011), ngoc-lien (11-01-2011), nguyenvanphung (06-07-2011), thanh_kha (11-01-2011), Thuong_19 (05-03-2011)
Old 11-01-2011, 11:45 AM   #2
novae
+Thành Viên Danh Dự+
 
novae's Avatar
 
Tham gia ngày: Jul 2010
Đến từ: Event horizon
Bài gởi: 2,453
Thanks: 53
Thanked 3,057 Times in 1,288 Posts
Đề Thi Chọn HSGQG Năm 2011 - Môn Toán Học
Thời gian làm bài 180 phút
Ngày thi thứ nhất 11/1/2011

[Only registered and activated users can see links. ] (5.0 điểm)
Cho $x $ là số thực dương và $n $ là số nguyên dương. Chứng minh bất đẳng thức:
$\frac{x^n \left( x^{n+1}+1 \right)}{x^n+1} \le \left( \frac{x+1}{2} \right)^{2n+1} $
Đẳng thức xảy ra khi nào?

[Only registered and activated users can see links. ] (5.0 điểm)
Cho dãy $\{x_n\} $ được xác định bởi:
$x_1=1;x_n=\frac{2n}{(n-1)^2} \sum_{i=1}^{n-1} x_i $
Chứng minh rằng dãy $y_n=x_{n+1}-x_n $ có giới hạn hữu hạn khi $n\to+\infty $

[Only registered and activated users can see links. ] (5.0 điểm)
Cho đường tròn $(O) $ đường kính $AB $. $P $ là một điểm trên tiếp tuyến của $(O) $ tại $B \; (P \ne B) $. Đường thẳng $AP $ cắt $(O) $ lần thứ hai tại $C $. $D $ là điểm đối xứng với $C $ qua $O $. Đường thẳng $DP $ cắt $(O) $ lần thứ hai tại $E $.
  1. Chứng minh rằng $AE,BC,PO $ đồng quy tại $M $
  2. Tìm vị trí của $P $ để diện tích tam giác $AMB $ lớn nhất. Tính diện tích lớn nhất đó theo $R $ là bán kính của $(O) $

[Only registered and activated users can see links. ] (5.0 điểm)
Cho ngũ giác lồi $ABCDE $ có các cạnh và 2 đường chéo $AC,AD $ có độ dài không vượt quá $\sqrt3 $. Trong ngũ giác lồi lấy $2011 $ điểm phân biệt bất kì. Chứng minh rằng tồn tại một hình tròn đơn vị có tâm nằm trên cạnh của ngũ giác lồi $ABCDE $ và chứa ít nhất $403 $ điểm trong số $2011 $ điểm đã cho

Chú ý:thí sinh không được sử dụng tài liệu nào khác hay máy tính cầm tay

------------------------------------------------------Hết ngày thi thứ nhất------------------------------------------------------




Đề Thi Chọn HSGQG Năm 2011 - Môn Toán Học

Thời gian làm bài 180 phút
Ngày thi thứ hai 12/1/2011



[Only registered and activated users can see links. ] (7 điểm):
Cho dãy số nguyên $(a_n) $ xác định bởi:
$a_0 =1; a_1=-1 $;

$a_n=6a_{n-1} + 5a_{n-2} $ với mọi $n \geq 2 $

Chứng minh rằng $a_{2012}-2010 $ chia hết cho $2011 $

[Only registered and activated users can see links. ] (7 điểm):
Cho tam giác $ABC $ không cân tại $A $ và có các góc $ABC $, $ACB $ là các góc nhọn. Xét 1 điểm $D $ di động trên cạnh $BC $ sao cho

$D $ không trùng với $B, C $ và hình chiếu vuông góc của A trên BC. Đường thẳng $d $ vuông góc với $BC $ tại $D $cắt đường thẳng $AB $,

$AC $ tương ứng tại $E $ và$ F $. Gọi $M,N $ và $P $ lần lượt là tâm đường tròn nội tiếp các tam giác $AEF, BDE $ và $CDF $. Chứng minh rằng

4 điểm $A, M, N, P $ cùng nằm trên một đường tròn khi và chỉ khi đường thẳng $d $ đi qua tâm đường tròn nội tiếp tam giác $ABC $


[Only registered and activated users can see links. ]: (6 điểm)
Cho n là số nguyên dương. Chứng minh rằng đa thức
$P(x,y) = x^n + xy + y^n $ không thể viết dưới dạng

$P(x,y) = G(x,y).H(x,y) $

Trong đó $G(x,y) $ và $H(x,y) $ là các đa thức với hệ số thực, khác đa thức hằng.

------------------------------------Hết-----------------------------
[RIGHT][I][B]Nguồn: MathScope.ORG[/B][/I][/RIGHT]
 
__________________
M.

thay đổi nội dung bởi: novae, 14-03-2011 lúc 08:36 PM Lý do: Lời giải bài 5
novae is offline   Trả Lời Với Trích Dẫn
The Following 70 Users Say Thank You to novae For This Useful Post:
bboy114crew (12-06-2011), bichlien31 (12-01-2011), binladen93 (11-01-2011), bluesday (12-01-2011), boconganh (13-01-2011), cattuong (12-01-2011), cuong_kimvan (11-01-2011), Curi_Gem (11-01-2011), cvppro (11-01-2011), daylight (11-01-2011), death_vs_love (06-03-2011), dep_kom_n (11-01-2011), ducadongnoi (05-03-2011), Duccuong94 (12-01-2011), ductho (14-01-2011), Dungmathscope (12-01-2011), duycvp (12-01-2011), einstein2205 (25-02-2011), ghetvan (18-01-2012), hizact (12-01-2011), hnhuongcoi (12-01-2011), hoangkhtn2010 (11-01-2011), huynhcongbang (12-01-2011), IMO 2010 (13-01-2011), inhtoan (12-01-2011), Ino_chan (06-03-2011), je.triste (25-02-2011), khicon (11-01-2011), kiengnho (11-01-2011), kiffen14 (11-01-2011), kryptios (12-01-2011), kuma (12-01-2011), lady_kom4 (11-01-2011), leedt26 (16-07-2011), lehuu (12-01-2011), leonaole94 (08-02-2011), levantuyen (04-07-2011), lexuanthang (12-01-2011), Lil.Tee (16-04-2011), linh_kc (20-02-2011), lk_95 (11-01-2011), louisgvr (12-01-2011), luatdhv (11-01-2011), lucatony1234 (13-01-2011), manhnguyen94 (11-01-2011), MathForLife (11-01-2011), Mệnh Thiên Tử (11-01-2011), Member_Of_AMC (12-01-2011), modular (14-01-2011), n.v.thanh (11-01-2011), nam1994 (11-01-2011), ngoc-lien (11-01-2011), nhox12764 (12-01-2011), nongdenchet (11-01-2011), polmki (11-01-2011), sandinh1308 (11-01-2011), Sang_kalot (11-01-2011), shinomoriaoshi (11-01-2011), stupidboy (12-01-2011), super_virus (11-02-2011), thanhquang0410 (12-01-2011), thanh_kha (11-01-2011), Thuong_19 (05-03-2011), tranvuxuannhat (12-01-2011), truongvoki_bn (11-01-2011), TSBT (11-01-2011), tuankhoa (11-01-2011), view (12-01-2011), vthiep94 (11-01-2011), winwave (11-01-2011)
Old 11-01-2011, 11:53 AM   #3
khicon
+Thành Viên+
 
Tham gia ngày: Aug 2009
Bài gởi: 73
Thanks: 77
Thanked 19 Times in 14 Posts
Có ai làm được bài tổ hợp không ạ???
em thấy bài này khá cơ bản, không biết hướng làm như thế nào?
------------------------------
Bài 1 có lẽ là chứng minh quy nạp, bài 2 chứng minh dãy tăng bị chặn trên bởi 4
không biết có đúng không nhỉ????
[RIGHT][I][B]Nguồn: MathScope.ORG[/B][/I][/RIGHT]
 

thay đổi nội dung bởi: khicon, 11-01-2011 lúc 11:54 AM Lý do: Tự động gộp bài
khicon is offline   Trả Lời Với Trích Dẫn
The Following 2 Users Say Thank You to khicon For This Useful Post:
Ino_chan (06-03-2011), tanglangquan (17-01-2011)
Old 11-01-2011, 11:56 AM   #4
alibaba_cqt
+Thành Viên+
 
alibaba_cqt's Avatar
 
Tham gia ngày: Sep 2010
Bài gởi: 255
Thanks: 42
Thanked 445 Times in 186 Posts
Đây là bản chính thức của đề thi. Qua Scan nhưng vẫn đọc khá rõ.
[RIGHT][I][B]Nguồn: MathScope.ORG[/B][/I][/RIGHT]
 
File Kèm Theo
Kiểu File : pdf De thi HSG Quoc gia 2010-2011.pdf (782.8 KB, 1235 lần tải)
__________________
$-1=(-1)^3=(-1)^{\frac{6}{2}}=(-1)^{6.\frac{1}{2}}=\left [(-1)^6 \right ]^{\frac{1}{2}}=1^{\frac{1}{2}}=1 $

http://www.youtube.com/watch?v=HVeQAuI3BQQ

thay đổi nội dung bởi: alibaba_cqt, 11-01-2011 lúc 03:36 PM
alibaba_cqt is offline   Trả Lời Với Trích Dẫn
The Following 10 Users Say Thank You to alibaba_cqt For This Useful Post:
caubemetoan96 (10-11-2012), cuong_kimvan (11-01-2011), huynhcongbang (12-01-2011), IMO 2010 (13-01-2011), lk_95 (11-01-2011), lucatony1234 (13-01-2011), n.v.thanh (11-01-2011), nhox12764 (12-01-2011), tanglangquan (17-01-2011), Unknowing (11-01-2011)
Old 11-01-2011, 11:56 AM   #5
novae
+Thành Viên Danh Dự+
 
novae's Avatar
 
Tham gia ngày: Jul 2010
Đến từ: Event horizon
Bài gởi: 2,453
Thanks: 53
Thanked 3,057 Times in 1,288 Posts
Trích:
Nguyên văn bởi khicon View Post
Có ai làm được bài tổ hợp không ạ???
em thấy bài này khá cơ bản, không biết hướng làm như thế nào?
------------------------------
Bài 1 có lẽ là chứng minh quy nạp, bài 2 chứng minh dãy tăng bị chặn trên bởi 4
không biết có đúng không nhỉ????
Bài tổ hợp chứng minh 1 tam giác có 3 cạnh $\le \sqrt3 $ được phủ bởi 3 hình tròn đơn vị tâm tại đỉnh là xong
[RIGHT][I][B]Nguồn: MathScope.ORG[/B][/I][/RIGHT]
 
__________________
M.

thay đổi nội dung bởi: novae, 11-01-2011 lúc 12:18 PM
novae is offline   Trả Lời Với Trích Dẫn
The Following 3 Users Say Thank You to novae For This Useful Post:
IMO 2010 (13-01-2011), Ino_chan (06-03-2011), mrcool (13-01-2011)
Old 11-01-2011, 11:59 AM   #6
huy2710
+Thành Viên+
 
Tham gia ngày: Apr 2009
Bài gởi: 20
Thanks: 1
Thanked 2 Times in 2 Posts
Trích:
Nguyên văn bởi novae View Post
Bài tổ hợp chứng minh 1 tam giác có 3 cạnh $\le 3 $ được phủ bởi 3 hình tròn đơn vị tâm tại đỉnh là xong
$\sqrt{3} $ chứ bạn. Mình cũng làm như vậy, phản chứng có tốt đa 402 điểm thuộc mỗi đường tròn, do đó cần ít nhất 6 đường tròn trong đó 5 đường tròn không thể phủ hết ngũ giác. Chứng minh cái của bạn nữa là xong

uh, mình gõ thiếu chút
[RIGHT][I][B]Nguồn: MathScope.ORG[/B][/I][/RIGHT]
 

thay đổi nội dung bởi: novae, 11-01-2011 lúc 12:18 PM
huy2710 is offline   Trả Lời Với Trích Dẫn
The Following User Says Thank You to huy2710 For This Useful Post:
Ino_chan (06-03-2011)
Old 11-01-2011, 12:00 PM   #7
lion
+Thành Viên Danh Dự+
 
lion's Avatar
 
Tham gia ngày: Apr 2010
Đến từ: Hà Nội
Bài gởi: 389
Thanks: 67
Thanked 133 Times in 97 Posts
Trích:
Nguyên văn bởi novae View Post
Bài tổ hợp chứng minh 1 tam giác có 3 cạnh $\le 3 $ được phủ bởi 3 hình tròn đơn vị tâm tại đỉnh là xong
Nhưng 5 đường tròn tâm tại 5 đỉnh bán kính 1 không phủ hết ngũ giác!!!
[RIGHT][I][B]Nguồn: MathScope.ORG[/B][/I][/RIGHT]
 
__________________
Đã trở lại
lion is offline   Trả Lời Với Trích Dẫn
Old 11-01-2011, 12:01 PM   #8
hophinhan_LHP
+Thành Viên+
 
hophinhan_LHP's Avatar
 
Tham gia ngày: Dec 2008
Đến từ: 12CT_THPT Chuyên LHP_TPHCM
Bài gởi: 226
Thanks: 199
Thanked 136 Times in 81 Posts
Trích:
Nguyên văn bởi sonltv_94 View Post
Sax câu hình 3a là Pascal suy biến
chú ý trục đẳng phương của (PAB) voi (PCD)
[RIGHT][I][B]Nguồn: MathScope.ORG[/B][/I][/RIGHT]
 
__________________
ĐẠI HỌC THÔI !!!

thay đổi nội dung bởi: hophinhan_LHP, 11-01-2011 lúc 12:06 PM
hophinhan_LHP is offline   Trả Lời Với Trích Dẫn
Old 11-01-2011, 12:02 PM   #9
nguyenmackhai
+Thành Viên+
 
Tham gia ngày: Sep 2008
Bài gởi: 52
Thanks: 18
Thanked 14 Times in 8 Posts
Thất bại vì mình quá kém. Hic. Cho mình hỏi bài 2 các bạn làm ntn?
[RIGHT][I][B]Nguồn: MathScope.ORG[/B][/I][/RIGHT]
 
nguyenmackhai is offline   Trả Lời Với Trích Dẫn
The Following User Says Thank You to nguyenmackhai For This Useful Post:
hoangnamb3 (12-01-2011)
Old 11-01-2011, 12:05 PM   #10
n.v.thanh
Moderator
 
n.v.thanh's Avatar
 
Tham gia ngày: Nov 2009
Bài gởi: 2,849
Thanks: 2,980
Thanked 2,537 Times in 1,008 Posts
Bài hình dễ quá.
Cấu hình này mấy đề thi vào lớp 10 cho mãi rồi.

Ý 1:Kéo dài $AE $ cắt $PB $ ở $I $.Dễ có $CI $ song song $AB $(tứ giác $PCEI,ACEB $ nội tiếp)

Ý 2 Xét tỉ số $\dfrac{S_{MAB}}{S_{PAB}} =\frac{OM}{OP}=\frac{CA}{CA+2CP} $


Dưới mẫu AM-GM:

$CA+2CP\geq 2\sqrt{CA.2CP}=BC.2.\sqrt2 $

$ S_{PAB}=\frac{1}{2}PC.BC $

Đáp số $\frac{R^2}{\sqrt 2} $

Vị trí P sao cho$PB=R.2\sqrt2 $
Hứa hẹn ngày mai.Không biết là tổ hợp hay hình câu cuối do cả câu 2,4 hôm nay đều chưa phải tầm của câu hình hay tổ hợp khó nhất đề.Mong là tổ hợp cho chết cùng nhau
Tưởng tượng nên có thể sai vị trí điểm
[RIGHT][I][B]Nguồn: MathScope.ORG[/B][/I][/RIGHT]
 

thay đổi nội dung bởi: n.v.thanh, 11-01-2011 lúc 12:57 PM
n.v.thanh is offline   Trả Lời Với Trích Dẫn
The Following 9 Users Say Thank You to n.v.thanh For This Useful Post:
binladen93 (11-01-2011), bluesday (11-01-2011), Curi_Gem (11-01-2011), hoangkhtn2010 (11-01-2011), IMO 2010 (13-01-2011), Ino_chan (06-03-2011), lk_95 (11-01-2011), lucatony1234 (13-01-2011), tranvuxuannhat (11-01-2011)
Old 11-01-2011, 12:05 PM   #11
huy2710
+Thành Viên+
 
Tham gia ngày: Apr 2009
Bài gởi: 20
Thanks: 1
Thanked 2 Times in 2 Posts
Trích:
Nguyên văn bởi lion View Post
Nhưng 5 đường tròn tâm tại 5 đỉnh bán kính 1 không phủ hết ngũ giác!!!
Đề cho các cạnh k vượt quá $\sqrt{3} $ mà bạn, mỗi hình tròn có bán kính là $1>\frac{\sqrt{3}}{2} $ rồi
[RIGHT][I][B]Nguồn: MathScope.ORG[/B][/I][/RIGHT]
 
huy2710 is offline   Trả Lời Với Trích Dẫn
Old 11-01-2011, 12:06 PM   #12
can_hang2008
+Thành Viên+
 
Tham gia ngày: Mar 2009
Bài gởi: 310
Thanks: 5
Thanked 751 Times in 187 Posts
Bài 1 khá cũ rồi. Bài gốc của nó là kết quả của bài BMO 2008:

Cho $a,\;b $ là các số thực dương thỏa mãn $a+b=2. $ Với mỗi $n \in \mathbb N^*, $ tìm tất cả các giả trị $m \in \mathbb N^* $ sao cho
$\frac{1}{a^n}+\frac{1}{b^n} \ge a^m+b^m. $

[RIGHT][I][B]Nguồn: MathScope.ORG[/B][/I][/RIGHT]
 
__________________
The love makes us stronger!

Võ Quốc Bá Cẩn
can_hang2008 is offline   Trả Lời Với Trích Dẫn
The Following 18 Users Say Thank You to can_hang2008 For This Useful Post:
Curi_Gem (11-01-2011), gxstar (16-01-2011), hien123 (11-01-2011), hoangkhtn2010 (11-01-2011), IMO 2010 (13-01-2011), Ino_chan (06-03-2011), kiffen14 (12-01-2011), lehuu (12-01-2011), letientai (11-01-2011), lk_95 (11-01-2011), luatdhv (11-01-2011), lucatony1234 (13-01-2011), mrcool (12-01-2011), n.v.thanh (11-01-2011), nhox12764 (12-01-2011), tranvuxuannhat (11-01-2011), vjpbozz (12-01-2011), zớt (31-07-2012)
Old 11-01-2011, 12:11 PM   #13
khicon
+Thành Viên+
 
Tham gia ngày: Aug 2009
Bài gởi: 73
Thanks: 77
Thanked 19 Times in 14 Posts
Trích:
Nguyên văn bởi novae View Post
Bài tổ hợp chứng minh 1 tam giác có 3 cạnh $\le 3 $ được phủ bởi 3 hình tròn đơn vị tâm tại đỉnh là xong
thế nếu chứng minh theo hướng 5 đường tròn tâm là 5 trung điểm của 5 cạnh thì sao ạ??
em chứng minh 1 tứ giác có cạnh và 1 đường chéo nhỏ hơn hoặc bằng căn 3 thì bị phủ kín bởi 3 đường tròn đơn vị tâm là 3 trung điểm 3 cạnh, nhưng chưa chứng minh được trọn vẹn. không biết hướng này có đúng không???
[RIGHT][I][B]Nguồn: MathScope.ORG[/B][/I][/RIGHT]
 
khicon is offline   Trả Lời Với Trích Dẫn
Old 11-01-2011, 12:14 PM   #14
ThangToan
+Thành Viên+
 
Tham gia ngày: Nov 2010
Đến từ: THPT chuyên Vĩnh Phúc
Bài gởi: 570
Thanks: 24
Thanked 537 Times in 263 Posts
Câu 1. Chứng minh bằng quy nạp:
Ta có $(\frac{x+1}{2})^{2n+3}=(\frac{x+1}{2})^{2n+1} (\frac{x+1}{2})^2\ge \frac{x^{n+1}(x^{n+2}+1)}{x^{n+1}+1}.(\frac{x+1}{2 })^2 $.
Ta sẽ chứngminh $\frac{x^{n+1}(x^{n+2}+1)}{x^{n+1}+1}.(\frac{x+1}{2 })^2\ge \frac{x^{n}(x^{n+1}+1)}{x^{n}+1}.(\frac{x+1}{2})^2 $ bất đẳng thức này tương đương với: $(x^{n+1}-1)^2(x-1)^2\ge 0 $
[RIGHT][I][B]Nguồn: MathScope.ORG[/B][/I][/RIGHT]
 

thay đổi nội dung bởi: ThangToan, 11-01-2011 lúc 12:17 PM
ThangToan is offline   Trả Lời Với Trích Dẫn
The Following 14 Users Say Thank You to ThangToan For This Useful Post:
binladen93 (11-01-2011), cattuong (11-01-2011), cong tien ly (12-02-2011), Curi_Gem (11-01-2011), hoangkhtn2010 (12-01-2011), huynhcongbang (12-01-2011), IMO 2010 (13-01-2011), khicon (11-01-2011), kuma (11-01-2011), lehuu (12-01-2011), lk_95 (11-01-2011), lucatony1234 (13-01-2011), n.v.thanh (11-01-2011), nqt (12-01-2011)
Old 11-01-2011, 12:16 PM   #15
khicon
+Thành Viên+
 
Tham gia ngày: Aug 2009
Bài gởi: 73
Thanks: 77
Thanked 19 Times in 14 Posts
Trích:
Nguyên văn bởi thangtoancvp View Post
Câu 1. Chứng minh bằng quy nạp:
Ta có $(\frac{x+1}{2})^{2n+3}=(\frac{x+1}{2})^{2n+1}(\fra c{x+1}{2})^2\ge \frac{x^{n+1}(x^{n+2}+1)}{x^{n+1}+1}.(\frac{x+1}{2 })^2 $.
Ta sẽ chứngminh $\frac{x^{n+1}(x^{n+2}+1)}{x^{n+1}+1}.(\frac{x+1}{2 })^2\ge \frac{x^{n}(x^{n+1}+1)}{x^{n}+1}.(\frac{x+1}{2})^2 $ bất đẳng thức này tương đương với: $(x^{n+1}-1)^2(x-1)^2\ge 0 $
uhm uhm, có lẽ đây là bài cho điểm
[RIGHT][I][B]Nguồn: MathScope.ORG[/B][/I][/RIGHT]
 
khicon is offline   Trả Lời Với Trích Dẫn
The Following User Says Thank You to khicon For This Useful Post:
vnmo (03-03-2011)
Trả lời Gởi Ðề Tài Mới

Bookmarks

Tags
2010-2011, hsg quốc gia

Ðiều Chỉnh
Xếp Bài

Quuyền Hạn Của Bạn
You may not post new threads
You may not post replies
You may not post attachments
You may not edit your posts

BB code is Mở
Smilies đang Mở
[IMG] đang Mở
HTML đang Tắt

Chuyển đến


Múi giờ GMT. Hiện tại là 10:26 AM.


Powered by: vBulletin Copyright ©2000-2024, Jelsoft Enterprises Ltd.
Inactive Reminders By mathscope.org
[page compression: 129.71 k/146.60 k (11.52%)]